Could someone please explain this?
Could someone please explain this? Thanks
jingjingxiao11111@gmail.com on November 22 at 09:16PM
  • October 2000 LSAT
  • SEC3
  • Q10
1
Reply
Why is C out?
We see R, and I in on Wednesday on the Board for Question 1...
cheynnelee on November 22 at 09:09PM
  • October 2000 LSAT
  • SEC3
  • Q3
1
Reply
Can we get a complete explanation of these games?
Can we get a complete explanation of the setup of this game as well as explanations for 7 - 11 ?
cheynnelee on August 5 at 10:08PM
  • October 2000 LSAT
  • SEC3
  • Q7
2
Replies
What Ross saw in the rules
In the video Ross says he doesn't see anything that lends itself to making scenarios which is a g...
AndrewArabie on February 5, 2023
  • October 2000 LSAT
  • SEC3
  • Q1
2
Replies
Can you please explain answer?
Hello, I was going over this last question and was wondering why b is wrong. Is it not possi...
mayatassi on March 20, 2022
  • October 2000 LSAT
  • SEC3
  • Q11
2
Replies
The last question for this prompt is wrong
Hi, I was doing this set of questions and the last question (The one I am messaging about) is a c...
Hayley-Lyon on June 23, 2021
  • October 2000 LSAT
  • SEC3
  • Q1
1
Reply
Why is E Out?
Can somone explain why E is out?
cheynnelee on January 28, 2021
  • October 2000 LSAT
  • SEC3
  • Q24
1
Reply
D
Why is D incorrect? Why couldn't you select three french novels and three russian novels?
MrLaw on May 8, 2020
  • October 2000 LSAT
  • SEC3
  • Q8
2
Replies
Rule #3
Can you explain the rule #3 "At least as many French novels....." I believe my misinterpretatio...
MGN2014 on August 18, 2014
  • October 2000 LSAT
  • SEC3
  • Q8
2
Replies
Difference between A and E?
I'm confused how would it be E instead of A.
MGN2014 on August 16, 2014
  • October 2000 LSAT
  • SEC3
  • Q15
1
Reply
At least terminology
I keep getting tripped up with the "at least" phrase that throws me off. Because I selected C bec...
MGN2014 on August 16, 2014
  • October 2000 LSAT
  • SEC3
  • Q14
3
Replies
Setup
Is it possible to view the set up of this game?
MGN2014 on August 14, 2014
  • October 2000 LSAT
  • SEC3
  • Q3
1
Reply